JEE Main & Advanced Mathematics Sequence & Series Question Bank Exponential series

  • question_answer
    \[\frac{2}{1\,!}+\frac{2+4}{2\,!}+\frac{2+4+6}{3\,!}+....\infty =\]  [MNR 1985]

    A) \[e\]

    B) \[2\,e\]

    C) \[3\,e\]

    D) None of these

    Correct Answer: C

    Solution :

      \[S=\frac{2}{1\ !}+\frac{2+4}{2\ !}+\frac{2+4+6}{3\ !}+......+\frac{\frac{n}{2}(2+2n)}{n\ !}+.....\infty \] Here \[{{T}_{n}}=\frac{n(n+1)}{n\ !}=\frac{n-1+2}{(n-1)\ !}=\frac{1}{(n-2)\ !}+\frac{2}{(n-1)\ !}\] \[\Rightarrow S=\sum\limits_{n=1}^{\infty }{{{T}_{n}}}=\sum\limits_{n=1}^{\infty }{\frac{1}{(n-2)\ !}+2\sum\limits_{n=1}^{\infty }{\frac{1}{(n-1)\ !}}}\]       \[=e+2e=3e\].


You need to login to perform this action.
You will be redirected in 3 sec spinner